Relativistic resistance transformationHow do moving charges produce magnetic fields?Relativistic origin of magnetic fieldConductor resistance calculation methodWhat is general strategy to find the resistance of medium?Why do we prefer using materials of high resistivity in laboratory instruments?total resistance of sector of cylinderResistivity of non-ohmic resistorIf Resistivity = $fracRAL$, why does it not depend on dimension?

Is there any algorithm that runs faster in Mathematica than in C or FORTRAN?

Where are the other 36 children born the same day?

How to determine which sectors a file occupies on Windows?

Is there a more elegant way to express ((x == a and y == b) or (x == b and y == a))?

"Du hast es gut", small talk meaning?

Why do some web servers still provide information on vendor and version in the HTTP response headers

Why is double encryption that's equivalent to single encryption no better than single encryption?

Countering an uncounterable spell, triggers, and state-based actions

True or False: If the product of n elements of a group is the identity element, it remains so no matter in what order the terms are multiplied.

Covering an 8x8 grid with X pentominoes

My cat gets angry and scared at me if I stand

What is the basis for the custom to stand or sit by Vayivarech Dovid?

How do I assign the Output of a command as a Value of a variable?

The Best™ class

Company asks (more than once) if I can involve family members in project

As tourist in China do I have to fear consequences for having publicly liked South Park?

Would the minimum payment or full CC amount be considered monthly debt?

What's the most profitable use for an elemental transmuter?

Does animal blood, esp. human, really have similar salinity as ocean water, and does that prove anything about evolution?

Can you put L trominos to fill the figure?

Why do Climate activists attack public transport?

Letters associated with prime numbers

A word/phrase means "a small amount" (of a color)

Can a company prevent a co-author of a paper from putting his name on it?



Relativistic resistance transformation


How do moving charges produce magnetic fields?Relativistic origin of magnetic fieldConductor resistance calculation methodWhat is general strategy to find the resistance of medium?Why do we prefer using materials of high resistivity in laboratory instruments?total resistance of sector of cylinderResistivity of non-ohmic resistorIf Resistivity = $fracRAL$, why does it not depend on dimension?






.everyoneloves__top-leaderboard:empty,.everyoneloves__mid-leaderboard:empty,.everyoneloves__bot-mid-leaderboard:empty
margin-bottom:0;









9














$begingroup$


During a discussion with my professor, he asked me about relativistic resistance transformation. Firstly I started with the formula $R=fracrho Ls$, where $rho$ is electric resistivity. So if some resistor moves with relativistic speed its length contracts. And this means that its resistance changes like its length. Its resistivity doesn't change because of Ohm's law $E=rho I$. Current changes like an electric field and that means that $rho=textconst$. But then the professor said that to find this transformation I can't use the first formula, and I should find it from some laws that have resistance in it. So I used $$P=I^2R tag2$$ and $$R=fracUI tag3.$$ From formula (3) we find that resistance doesn't change because difference of potentials and current change the same way. But from formula (2) we find that resistance changes like $fracRgamma^3 $ (where $gamma$ is Lorentz factor). So basically I have three variants, but the professor said that I need to find one result. Any ideas?










share|cite|improve this question












$endgroup$














  • $begingroup$
    Lots of things change with reference frame in EM. See : Relativistic origin of magnetic field, How do moving charges produce magnetic fields?
    $endgroup$
    – J...
    May 30 at 19:10










  • $begingroup$
    All the components of your circuit will be moving at the same velocity, how would you see relativistic effects?
    $endgroup$
    – Loren Pechtel
    May 31 at 1:31

















9














$begingroup$


During a discussion with my professor, he asked me about relativistic resistance transformation. Firstly I started with the formula $R=fracrho Ls$, where $rho$ is electric resistivity. So if some resistor moves with relativistic speed its length contracts. And this means that its resistance changes like its length. Its resistivity doesn't change because of Ohm's law $E=rho I$. Current changes like an electric field and that means that $rho=textconst$. But then the professor said that to find this transformation I can't use the first formula, and I should find it from some laws that have resistance in it. So I used $$P=I^2R tag2$$ and $$R=fracUI tag3.$$ From formula (3) we find that resistance doesn't change because difference of potentials and current change the same way. But from formula (2) we find that resistance changes like $fracRgamma^3 $ (where $gamma$ is Lorentz factor). So basically I have three variants, but the professor said that I need to find one result. Any ideas?










share|cite|improve this question












$endgroup$














  • $begingroup$
    Lots of things change with reference frame in EM. See : Relativistic origin of magnetic field, How do moving charges produce magnetic fields?
    $endgroup$
    – J...
    May 30 at 19:10










  • $begingroup$
    All the components of your circuit will be moving at the same velocity, how would you see relativistic effects?
    $endgroup$
    – Loren Pechtel
    May 31 at 1:31













9












9








9


3



$begingroup$


During a discussion with my professor, he asked me about relativistic resistance transformation. Firstly I started with the formula $R=fracrho Ls$, where $rho$ is electric resistivity. So if some resistor moves with relativistic speed its length contracts. And this means that its resistance changes like its length. Its resistivity doesn't change because of Ohm's law $E=rho I$. Current changes like an electric field and that means that $rho=textconst$. But then the professor said that to find this transformation I can't use the first formula, and I should find it from some laws that have resistance in it. So I used $$P=I^2R tag2$$ and $$R=fracUI tag3.$$ From formula (3) we find that resistance doesn't change because difference of potentials and current change the same way. But from formula (2) we find that resistance changes like $fracRgamma^3 $ (where $gamma$ is Lorentz factor). So basically I have three variants, but the professor said that I need to find one result. Any ideas?










share|cite|improve this question












$endgroup$




During a discussion with my professor, he asked me about relativistic resistance transformation. Firstly I started with the formula $R=fracrho Ls$, where $rho$ is electric resistivity. So if some resistor moves with relativistic speed its length contracts. And this means that its resistance changes like its length. Its resistivity doesn't change because of Ohm's law $E=rho I$. Current changes like an electric field and that means that $rho=textconst$. But then the professor said that to find this transformation I can't use the first formula, and I should find it from some laws that have resistance in it. So I used $$P=I^2R tag2$$ and $$R=fracUI tag3.$$ From formula (3) we find that resistance doesn't change because difference of potentials and current change the same way. But from formula (2) we find that resistance changes like $fracRgamma^3 $ (where $gamma$ is Lorentz factor). So basically I have three variants, but the professor said that I need to find one result. Any ideas?







electromagnetism special-relativity electrical-resistance






share|cite|improve this question
















share|cite|improve this question













share|cite|improve this question




share|cite|improve this question








edited Jun 6 at 14:28









Glorfindel

6681 gold badge9 silver badges15 bronze badges




6681 gold badge9 silver badges15 bronze badges










asked May 29 at 18:48









Semen YurchenkoSemen Yurchenko

1185 bronze badges




1185 bronze badges














  • $begingroup$
    Lots of things change with reference frame in EM. See : Relativistic origin of magnetic field, How do moving charges produce magnetic fields?
    $endgroup$
    – J...
    May 30 at 19:10










  • $begingroup$
    All the components of your circuit will be moving at the same velocity, how would you see relativistic effects?
    $endgroup$
    – Loren Pechtel
    May 31 at 1:31
















  • $begingroup$
    Lots of things change with reference frame in EM. See : Relativistic origin of magnetic field, How do moving charges produce magnetic fields?
    $endgroup$
    – J...
    May 30 at 19:10










  • $begingroup$
    All the components of your circuit will be moving at the same velocity, how would you see relativistic effects?
    $endgroup$
    – Loren Pechtel
    May 31 at 1:31















$begingroup$
Lots of things change with reference frame in EM. See : Relativistic origin of magnetic field, How do moving charges produce magnetic fields?
$endgroup$
– J...
May 30 at 19:10




$begingroup$
Lots of things change with reference frame in EM. See : Relativistic origin of magnetic field, How do moving charges produce magnetic fields?
$endgroup$
– J...
May 30 at 19:10












$begingroup$
All the components of your circuit will be moving at the same velocity, how would you see relativistic effects?
$endgroup$
– Loren Pechtel
May 31 at 1:31




$begingroup$
All the components of your circuit will be moving at the same velocity, how would you see relativistic effects?
$endgroup$
– Loren Pechtel
May 31 at 1:31










2 Answers
2






active

oldest

votes


















14
















$begingroup$

Repeating what @Dale said in a slightly different way, if you want relativistic laws, it is better to work with relativisitic quantities.



The response of matter to applied electromagnetic field is generally expressed through charge density ($rho$) and current density ($mathbfJ$). The 4D equivalent is the four-current:



$J^mu=left(crho, mathbfJright)^mu$



Next, the electric and magnetic fields are not covariant quantities in SR, so use the electromagnetic tensor: $F^munu$ (https://en.wikipedia.org/wiki/Electromagnetic_tensor)



Now, conductivity, which in general is a tensor, arises as a result of postulating a linear relationship between the applied electric field and the current density.



So we could say:



$J^mu=sigma^mu_nuetaF^nueta$



Now this general definition hoovers up pretty much all the simple types of electromagnetic response. But that's the point, what looks like conductivity response to one observer will look very different to another one.



You could postulate that in your rest frame ($barS$) the response is solely that of the isotropic conductivity ($sigma$), then the only non-zero components are ($c$ is the speed of light):



$barsigma^1_01=-barsigma_10=csigma/2$



$barsigma_02=-barsigma_20=csigma/2$



$barsigma_03=-barsigma^3_30=csigma/2$



Then consider the lab-frame relative to which the rest-frame is moving along x with speed $v$. You then transform your tensor $sigma^mu_etanu=fracpartial x^mupartial barx^alphafracpartial barx^betapartial x^etafracpartial barx^kappapartial x^nubarsigma^alpha_betakappa$. The only components to change are the ones involving x:



I get



$sigma^0_10=-sigma^0_01=left(fracvcright)gammacdotfraccsigma2$



$sigma^1_10=-sigma^1_01=gammacdotfraccsigma2$



So you could say that in lab-frame the conductivity in x direction will change by $gamma=1/sqrt1-left(v/cright)^2$. This is the second equation, but that's not the end of the story. There is the first equation, which in non-relativistic terms would lead to:



$rho=fracvsigmac^2gammacdot E_x$



Where $E_x$ is the x-component of the electric field. So you would see a build-up of charge density due to applied electric field.



So, as you now see, the picture is more complex than just finding an equation of resistance $R$






share|cite|improve this answer










$endgroup$














  • $begingroup$
    Clearly if you wanted resitance you would also have to integrate the whole thing in space, which would bring into play the length contraction.
    $endgroup$
    – Cryo
    May 29 at 23:53











  • $begingroup$
    I would add a factor $frac12$ in front of your relativistic equation (the one involving the conductivity tensor). This is usual when we contract two antisymetric tensors. This way, there's no factors of $1/2$ in the transformed conductivity. Also, please, consider adding a tag to your equations, using the LaTeX command tag#.
    $endgroup$
    – Cham
    May 30 at 2:13







  • 3




    $begingroup$
    In the case of an isotropic material, the conductivity tensor could be written explicitely using the material's four-velocity and the Minkowski metric: $$sigma_ab^c = sigma , (eta_a^c , u_b - eta_b^c , u_a).$$ Then: $$J^c = frac12 , sigma_ab^c , F^ab equiv sigma , F^cb , u_b,$$ which is Ohm's law in relativistic form (for an isotropic material of proper conductivity $sigma$). For a general four-velocity, this tensor gives $sigma_10^0 = sigma gamma , v$ and $sigma_10^1 = sigma gamma$ (using $c equiv 1$), as your formulae. No need for coords. transf.
    $endgroup$
    – Cham
    May 30 at 2:25











  • $begingroup$
    Very nice. I didn't have the patience to find the good form. Thank you
    $endgroup$
    – Cryo
    May 30 at 8:37










  • $begingroup$
    This is it. Expect things to become tensorial. When you see something acting weird like that, giving funny, inconsistent results in a context like this - sniff around - could be a tensor lurking.
    $endgroup$
    – The_Sympathizer
    May 30 at 17:00


















12
















$begingroup$


So basically I have three variants, but professor that I need to find one result. Any ideas?




Circuit theory is inherently nonrelativistic and so there is no relativistic circuit theory. This should actually not be surprising since circuit theory does not use the concept of space at all and relativity is a theory of spacetime.



The issue is in the assumptions that circuit theory is based on. The three assumptions are: (1) there is no net charge on any circuit element, (2) there is no magnetic flux outside of any circuit element, (3) the circuit is small so that electromagnetic influences can be assumed to propagate instantaneously.



Assumption (3) is expressly prohibited by relativity, and since a current density in one frame is both a current and a charge density in another then (1) will generally not be satisfied in all frames. Therefore, it is fatally flawed to attempt to find a relativistic version of Ohms law based on the equations of circuit theory since those laws are inherently non relativistic.



However, you could take the form of Ohm’s law that is based directly on Maxwell’s equations: $mathbf J = sigma mathbf E$. This law is based on Maxwell’s equations, which are completely relativistic. Since we know how $mathbf J$ and $mathbf E$ both transform then we can calculate how $sigma$ transforms.






share|cite|improve this answer












$endgroup$














  • $begingroup$
    J and E change the same way. So sigma doesn't change, it means R change like length?
    $endgroup$
    – Semen Yurchenko
    May 29 at 20:09






  • 6




    $begingroup$
    I would not recommend going from 𝜎 to 𝑅. Circuit theory is not relativistic so it doesn’t make sense to find transformation laws for quantities in circuit theory. Just leave it as 𝜎. By the way, J does not transform like E.
    $endgroup$
    – Dale
    May 29 at 20:37












Your Answer








StackExchange.ready(function()
var channelOptions =
tags: "".split(" "),
id: "151"
;
initTagRenderer("".split(" "), "".split(" "), channelOptions);

StackExchange.using("externalEditor", function()
// Have to fire editor after snippets, if snippets enabled
if (StackExchange.settings.snippets.snippetsEnabled)
StackExchange.using("snippets", function()
createEditor();
);

else
createEditor();

);

function createEditor()
StackExchange.prepareEditor(
heartbeatType: 'answer',
autoActivateHeartbeat: false,
convertImagesToLinks: false,
noModals: true,
showLowRepImageUploadWarning: true,
reputationToPostImages: null,
bindNavPrevention: true,
postfix: "",
imageUploader:
brandingHtml: "Powered by u003ca class="icon-imgur-white" href="https://imgur.com/"u003eu003c/au003e",
contentPolicyHtml: "User contributions licensed under u003ca href="https://creativecommons.org/licenses/by-sa/4.0/"u003ecc by-sa 4.0 with attribution requiredu003c/au003e u003ca href="https://stackoverflow.com/legal/content-policy"u003e(content policy)u003c/au003e",
allowUrls: true
,
noCode: true, onDemand: true,
discardSelector: ".discard-answer"
,immediatelyShowMarkdownHelp:true
);



);














draft saved

draft discarded
















StackExchange.ready(
function ()
StackExchange.openid.initPostLogin('.new-post-login', 'https%3a%2f%2fphysics.stackexchange.com%2fquestions%2f483190%2frelativistic-resistance-transformation%23new-answer', 'question_page');

);

Post as a guest















Required, but never shown


























2 Answers
2






active

oldest

votes








2 Answers
2






active

oldest

votes









active

oldest

votes






active

oldest

votes









14
















$begingroup$

Repeating what @Dale said in a slightly different way, if you want relativistic laws, it is better to work with relativisitic quantities.



The response of matter to applied electromagnetic field is generally expressed through charge density ($rho$) and current density ($mathbfJ$). The 4D equivalent is the four-current:



$J^mu=left(crho, mathbfJright)^mu$



Next, the electric and magnetic fields are not covariant quantities in SR, so use the electromagnetic tensor: $F^munu$ (https://en.wikipedia.org/wiki/Electromagnetic_tensor)



Now, conductivity, which in general is a tensor, arises as a result of postulating a linear relationship between the applied electric field and the current density.



So we could say:



$J^mu=sigma^mu_nuetaF^nueta$



Now this general definition hoovers up pretty much all the simple types of electromagnetic response. But that's the point, what looks like conductivity response to one observer will look very different to another one.



You could postulate that in your rest frame ($barS$) the response is solely that of the isotropic conductivity ($sigma$), then the only non-zero components are ($c$ is the speed of light):



$barsigma^1_01=-barsigma_10=csigma/2$



$barsigma_02=-barsigma_20=csigma/2$



$barsigma_03=-barsigma^3_30=csigma/2$



Then consider the lab-frame relative to which the rest-frame is moving along x with speed $v$. You then transform your tensor $sigma^mu_etanu=fracpartial x^mupartial barx^alphafracpartial barx^betapartial x^etafracpartial barx^kappapartial x^nubarsigma^alpha_betakappa$. The only components to change are the ones involving x:



I get



$sigma^0_10=-sigma^0_01=left(fracvcright)gammacdotfraccsigma2$



$sigma^1_10=-sigma^1_01=gammacdotfraccsigma2$



So you could say that in lab-frame the conductivity in x direction will change by $gamma=1/sqrt1-left(v/cright)^2$. This is the second equation, but that's not the end of the story. There is the first equation, which in non-relativistic terms would lead to:



$rho=fracvsigmac^2gammacdot E_x$



Where $E_x$ is the x-component of the electric field. So you would see a build-up of charge density due to applied electric field.



So, as you now see, the picture is more complex than just finding an equation of resistance $R$






share|cite|improve this answer










$endgroup$














  • $begingroup$
    Clearly if you wanted resitance you would also have to integrate the whole thing in space, which would bring into play the length contraction.
    $endgroup$
    – Cryo
    May 29 at 23:53











  • $begingroup$
    I would add a factor $frac12$ in front of your relativistic equation (the one involving the conductivity tensor). This is usual when we contract two antisymetric tensors. This way, there's no factors of $1/2$ in the transformed conductivity. Also, please, consider adding a tag to your equations, using the LaTeX command tag#.
    $endgroup$
    – Cham
    May 30 at 2:13







  • 3




    $begingroup$
    In the case of an isotropic material, the conductivity tensor could be written explicitely using the material's four-velocity and the Minkowski metric: $$sigma_ab^c = sigma , (eta_a^c , u_b - eta_b^c , u_a).$$ Then: $$J^c = frac12 , sigma_ab^c , F^ab equiv sigma , F^cb , u_b,$$ which is Ohm's law in relativistic form (for an isotropic material of proper conductivity $sigma$). For a general four-velocity, this tensor gives $sigma_10^0 = sigma gamma , v$ and $sigma_10^1 = sigma gamma$ (using $c equiv 1$), as your formulae. No need for coords. transf.
    $endgroup$
    – Cham
    May 30 at 2:25











  • $begingroup$
    Very nice. I didn't have the patience to find the good form. Thank you
    $endgroup$
    – Cryo
    May 30 at 8:37










  • $begingroup$
    This is it. Expect things to become tensorial. When you see something acting weird like that, giving funny, inconsistent results in a context like this - sniff around - could be a tensor lurking.
    $endgroup$
    – The_Sympathizer
    May 30 at 17:00















14
















$begingroup$

Repeating what @Dale said in a slightly different way, if you want relativistic laws, it is better to work with relativisitic quantities.



The response of matter to applied electromagnetic field is generally expressed through charge density ($rho$) and current density ($mathbfJ$). The 4D equivalent is the four-current:



$J^mu=left(crho, mathbfJright)^mu$



Next, the electric and magnetic fields are not covariant quantities in SR, so use the electromagnetic tensor: $F^munu$ (https://en.wikipedia.org/wiki/Electromagnetic_tensor)



Now, conductivity, which in general is a tensor, arises as a result of postulating a linear relationship between the applied electric field and the current density.



So we could say:



$J^mu=sigma^mu_nuetaF^nueta$



Now this general definition hoovers up pretty much all the simple types of electromagnetic response. But that's the point, what looks like conductivity response to one observer will look very different to another one.



You could postulate that in your rest frame ($barS$) the response is solely that of the isotropic conductivity ($sigma$), then the only non-zero components are ($c$ is the speed of light):



$barsigma^1_01=-barsigma_10=csigma/2$



$barsigma_02=-barsigma_20=csigma/2$



$barsigma_03=-barsigma^3_30=csigma/2$



Then consider the lab-frame relative to which the rest-frame is moving along x with speed $v$. You then transform your tensor $sigma^mu_etanu=fracpartial x^mupartial barx^alphafracpartial barx^betapartial x^etafracpartial barx^kappapartial x^nubarsigma^alpha_betakappa$. The only components to change are the ones involving x:



I get



$sigma^0_10=-sigma^0_01=left(fracvcright)gammacdotfraccsigma2$



$sigma^1_10=-sigma^1_01=gammacdotfraccsigma2$



So you could say that in lab-frame the conductivity in x direction will change by $gamma=1/sqrt1-left(v/cright)^2$. This is the second equation, but that's not the end of the story. There is the first equation, which in non-relativistic terms would lead to:



$rho=fracvsigmac^2gammacdot E_x$



Where $E_x$ is the x-component of the electric field. So you would see a build-up of charge density due to applied electric field.



So, as you now see, the picture is more complex than just finding an equation of resistance $R$






share|cite|improve this answer










$endgroup$














  • $begingroup$
    Clearly if you wanted resitance you would also have to integrate the whole thing in space, which would bring into play the length contraction.
    $endgroup$
    – Cryo
    May 29 at 23:53











  • $begingroup$
    I would add a factor $frac12$ in front of your relativistic equation (the one involving the conductivity tensor). This is usual when we contract two antisymetric tensors. This way, there's no factors of $1/2$ in the transformed conductivity. Also, please, consider adding a tag to your equations, using the LaTeX command tag#.
    $endgroup$
    – Cham
    May 30 at 2:13







  • 3




    $begingroup$
    In the case of an isotropic material, the conductivity tensor could be written explicitely using the material's four-velocity and the Minkowski metric: $$sigma_ab^c = sigma , (eta_a^c , u_b - eta_b^c , u_a).$$ Then: $$J^c = frac12 , sigma_ab^c , F^ab equiv sigma , F^cb , u_b,$$ which is Ohm's law in relativistic form (for an isotropic material of proper conductivity $sigma$). For a general four-velocity, this tensor gives $sigma_10^0 = sigma gamma , v$ and $sigma_10^1 = sigma gamma$ (using $c equiv 1$), as your formulae. No need for coords. transf.
    $endgroup$
    – Cham
    May 30 at 2:25











  • $begingroup$
    Very nice. I didn't have the patience to find the good form. Thank you
    $endgroup$
    – Cryo
    May 30 at 8:37










  • $begingroup$
    This is it. Expect things to become tensorial. When you see something acting weird like that, giving funny, inconsistent results in a context like this - sniff around - could be a tensor lurking.
    $endgroup$
    – The_Sympathizer
    May 30 at 17:00













14














14










14







$begingroup$

Repeating what @Dale said in a slightly different way, if you want relativistic laws, it is better to work with relativisitic quantities.



The response of matter to applied electromagnetic field is generally expressed through charge density ($rho$) and current density ($mathbfJ$). The 4D equivalent is the four-current:



$J^mu=left(crho, mathbfJright)^mu$



Next, the electric and magnetic fields are not covariant quantities in SR, so use the electromagnetic tensor: $F^munu$ (https://en.wikipedia.org/wiki/Electromagnetic_tensor)



Now, conductivity, which in general is a tensor, arises as a result of postulating a linear relationship between the applied electric field and the current density.



So we could say:



$J^mu=sigma^mu_nuetaF^nueta$



Now this general definition hoovers up pretty much all the simple types of electromagnetic response. But that's the point, what looks like conductivity response to one observer will look very different to another one.



You could postulate that in your rest frame ($barS$) the response is solely that of the isotropic conductivity ($sigma$), then the only non-zero components are ($c$ is the speed of light):



$barsigma^1_01=-barsigma_10=csigma/2$



$barsigma_02=-barsigma_20=csigma/2$



$barsigma_03=-barsigma^3_30=csigma/2$



Then consider the lab-frame relative to which the rest-frame is moving along x with speed $v$. You then transform your tensor $sigma^mu_etanu=fracpartial x^mupartial barx^alphafracpartial barx^betapartial x^etafracpartial barx^kappapartial x^nubarsigma^alpha_betakappa$. The only components to change are the ones involving x:



I get



$sigma^0_10=-sigma^0_01=left(fracvcright)gammacdotfraccsigma2$



$sigma^1_10=-sigma^1_01=gammacdotfraccsigma2$



So you could say that in lab-frame the conductivity in x direction will change by $gamma=1/sqrt1-left(v/cright)^2$. This is the second equation, but that's not the end of the story. There is the first equation, which in non-relativistic terms would lead to:



$rho=fracvsigmac^2gammacdot E_x$



Where $E_x$ is the x-component of the electric field. So you would see a build-up of charge density due to applied electric field.



So, as you now see, the picture is more complex than just finding an equation of resistance $R$






share|cite|improve this answer










$endgroup$



Repeating what @Dale said in a slightly different way, if you want relativistic laws, it is better to work with relativisitic quantities.



The response of matter to applied electromagnetic field is generally expressed through charge density ($rho$) and current density ($mathbfJ$). The 4D equivalent is the four-current:



$J^mu=left(crho, mathbfJright)^mu$



Next, the electric and magnetic fields are not covariant quantities in SR, so use the electromagnetic tensor: $F^munu$ (https://en.wikipedia.org/wiki/Electromagnetic_tensor)



Now, conductivity, which in general is a tensor, arises as a result of postulating a linear relationship between the applied electric field and the current density.



So we could say:



$J^mu=sigma^mu_nuetaF^nueta$



Now this general definition hoovers up pretty much all the simple types of electromagnetic response. But that's the point, what looks like conductivity response to one observer will look very different to another one.



You could postulate that in your rest frame ($barS$) the response is solely that of the isotropic conductivity ($sigma$), then the only non-zero components are ($c$ is the speed of light):



$barsigma^1_01=-barsigma_10=csigma/2$



$barsigma_02=-barsigma_20=csigma/2$



$barsigma_03=-barsigma^3_30=csigma/2$



Then consider the lab-frame relative to which the rest-frame is moving along x with speed $v$. You then transform your tensor $sigma^mu_etanu=fracpartial x^mupartial barx^alphafracpartial barx^betapartial x^etafracpartial barx^kappapartial x^nubarsigma^alpha_betakappa$. The only components to change are the ones involving x:



I get



$sigma^0_10=-sigma^0_01=left(fracvcright)gammacdotfraccsigma2$



$sigma^1_10=-sigma^1_01=gammacdotfraccsigma2$



So you could say that in lab-frame the conductivity in x direction will change by $gamma=1/sqrt1-left(v/cright)^2$. This is the second equation, but that's not the end of the story. There is the first equation, which in non-relativistic terms would lead to:



$rho=fracvsigmac^2gammacdot E_x$



Where $E_x$ is the x-component of the electric field. So you would see a build-up of charge density due to applied electric field.



So, as you now see, the picture is more complex than just finding an equation of resistance $R$







share|cite|improve this answer













share|cite|improve this answer




share|cite|improve this answer










answered May 29 at 22:26









CryoCryo

1,2843 silver badges9 bronze badges




1,2843 silver badges9 bronze badges














  • $begingroup$
    Clearly if you wanted resitance you would also have to integrate the whole thing in space, which would bring into play the length contraction.
    $endgroup$
    – Cryo
    May 29 at 23:53











  • $begingroup$
    I would add a factor $frac12$ in front of your relativistic equation (the one involving the conductivity tensor). This is usual when we contract two antisymetric tensors. This way, there's no factors of $1/2$ in the transformed conductivity. Also, please, consider adding a tag to your equations, using the LaTeX command tag#.
    $endgroup$
    – Cham
    May 30 at 2:13







  • 3




    $begingroup$
    In the case of an isotropic material, the conductivity tensor could be written explicitely using the material's four-velocity and the Minkowski metric: $$sigma_ab^c = sigma , (eta_a^c , u_b - eta_b^c , u_a).$$ Then: $$J^c = frac12 , sigma_ab^c , F^ab equiv sigma , F^cb , u_b,$$ which is Ohm's law in relativistic form (for an isotropic material of proper conductivity $sigma$). For a general four-velocity, this tensor gives $sigma_10^0 = sigma gamma , v$ and $sigma_10^1 = sigma gamma$ (using $c equiv 1$), as your formulae. No need for coords. transf.
    $endgroup$
    – Cham
    May 30 at 2:25











  • $begingroup$
    Very nice. I didn't have the patience to find the good form. Thank you
    $endgroup$
    – Cryo
    May 30 at 8:37










  • $begingroup$
    This is it. Expect things to become tensorial. When you see something acting weird like that, giving funny, inconsistent results in a context like this - sniff around - could be a tensor lurking.
    $endgroup$
    – The_Sympathizer
    May 30 at 17:00
















  • $begingroup$
    Clearly if you wanted resitance you would also have to integrate the whole thing in space, which would bring into play the length contraction.
    $endgroup$
    – Cryo
    May 29 at 23:53











  • $begingroup$
    I would add a factor $frac12$ in front of your relativistic equation (the one involving the conductivity tensor). This is usual when we contract two antisymetric tensors. This way, there's no factors of $1/2$ in the transformed conductivity. Also, please, consider adding a tag to your equations, using the LaTeX command tag#.
    $endgroup$
    – Cham
    May 30 at 2:13







  • 3




    $begingroup$
    In the case of an isotropic material, the conductivity tensor could be written explicitely using the material's four-velocity and the Minkowski metric: $$sigma_ab^c = sigma , (eta_a^c , u_b - eta_b^c , u_a).$$ Then: $$J^c = frac12 , sigma_ab^c , F^ab equiv sigma , F^cb , u_b,$$ which is Ohm's law in relativistic form (for an isotropic material of proper conductivity $sigma$). For a general four-velocity, this tensor gives $sigma_10^0 = sigma gamma , v$ and $sigma_10^1 = sigma gamma$ (using $c equiv 1$), as your formulae. No need for coords. transf.
    $endgroup$
    – Cham
    May 30 at 2:25











  • $begingroup$
    Very nice. I didn't have the patience to find the good form. Thank you
    $endgroup$
    – Cryo
    May 30 at 8:37










  • $begingroup$
    This is it. Expect things to become tensorial. When you see something acting weird like that, giving funny, inconsistent results in a context like this - sniff around - could be a tensor lurking.
    $endgroup$
    – The_Sympathizer
    May 30 at 17:00















$begingroup$
Clearly if you wanted resitance you would also have to integrate the whole thing in space, which would bring into play the length contraction.
$endgroup$
– Cryo
May 29 at 23:53





$begingroup$
Clearly if you wanted resitance you would also have to integrate the whole thing in space, which would bring into play the length contraction.
$endgroup$
– Cryo
May 29 at 23:53













$begingroup$
I would add a factor $frac12$ in front of your relativistic equation (the one involving the conductivity tensor). This is usual when we contract two antisymetric tensors. This way, there's no factors of $1/2$ in the transformed conductivity. Also, please, consider adding a tag to your equations, using the LaTeX command tag#.
$endgroup$
– Cham
May 30 at 2:13





$begingroup$
I would add a factor $frac12$ in front of your relativistic equation (the one involving the conductivity tensor). This is usual when we contract two antisymetric tensors. This way, there's no factors of $1/2$ in the transformed conductivity. Also, please, consider adding a tag to your equations, using the LaTeX command tag#.
$endgroup$
– Cham
May 30 at 2:13





3




3




$begingroup$
In the case of an isotropic material, the conductivity tensor could be written explicitely using the material's four-velocity and the Minkowski metric: $$sigma_ab^c = sigma , (eta_a^c , u_b - eta_b^c , u_a).$$ Then: $$J^c = frac12 , sigma_ab^c , F^ab equiv sigma , F^cb , u_b,$$ which is Ohm's law in relativistic form (for an isotropic material of proper conductivity $sigma$). For a general four-velocity, this tensor gives $sigma_10^0 = sigma gamma , v$ and $sigma_10^1 = sigma gamma$ (using $c equiv 1$), as your formulae. No need for coords. transf.
$endgroup$
– Cham
May 30 at 2:25





$begingroup$
In the case of an isotropic material, the conductivity tensor could be written explicitely using the material's four-velocity and the Minkowski metric: $$sigma_ab^c = sigma , (eta_a^c , u_b - eta_b^c , u_a).$$ Then: $$J^c = frac12 , sigma_ab^c , F^ab equiv sigma , F^cb , u_b,$$ which is Ohm's law in relativistic form (for an isotropic material of proper conductivity $sigma$). For a general four-velocity, this tensor gives $sigma_10^0 = sigma gamma , v$ and $sigma_10^1 = sigma gamma$ (using $c equiv 1$), as your formulae. No need for coords. transf.
$endgroup$
– Cham
May 30 at 2:25













$begingroup$
Very nice. I didn't have the patience to find the good form. Thank you
$endgroup$
– Cryo
May 30 at 8:37




$begingroup$
Very nice. I didn't have the patience to find the good form. Thank you
$endgroup$
– Cryo
May 30 at 8:37












$begingroup$
This is it. Expect things to become tensorial. When you see something acting weird like that, giving funny, inconsistent results in a context like this - sniff around - could be a tensor lurking.
$endgroup$
– The_Sympathizer
May 30 at 17:00




$begingroup$
This is it. Expect things to become tensorial. When you see something acting weird like that, giving funny, inconsistent results in a context like this - sniff around - could be a tensor lurking.
$endgroup$
– The_Sympathizer
May 30 at 17:00













12
















$begingroup$


So basically I have three variants, but professor that I need to find one result. Any ideas?




Circuit theory is inherently nonrelativistic and so there is no relativistic circuit theory. This should actually not be surprising since circuit theory does not use the concept of space at all and relativity is a theory of spacetime.



The issue is in the assumptions that circuit theory is based on. The three assumptions are: (1) there is no net charge on any circuit element, (2) there is no magnetic flux outside of any circuit element, (3) the circuit is small so that electromagnetic influences can be assumed to propagate instantaneously.



Assumption (3) is expressly prohibited by relativity, and since a current density in one frame is both a current and a charge density in another then (1) will generally not be satisfied in all frames. Therefore, it is fatally flawed to attempt to find a relativistic version of Ohms law based on the equations of circuit theory since those laws are inherently non relativistic.



However, you could take the form of Ohm’s law that is based directly on Maxwell’s equations: $mathbf J = sigma mathbf E$. This law is based on Maxwell’s equations, which are completely relativistic. Since we know how $mathbf J$ and $mathbf E$ both transform then we can calculate how $sigma$ transforms.






share|cite|improve this answer












$endgroup$














  • $begingroup$
    J and E change the same way. So sigma doesn't change, it means R change like length?
    $endgroup$
    – Semen Yurchenko
    May 29 at 20:09






  • 6




    $begingroup$
    I would not recommend going from 𝜎 to 𝑅. Circuit theory is not relativistic so it doesn’t make sense to find transformation laws for quantities in circuit theory. Just leave it as 𝜎. By the way, J does not transform like E.
    $endgroup$
    – Dale
    May 29 at 20:37















12
















$begingroup$


So basically I have three variants, but professor that I need to find one result. Any ideas?




Circuit theory is inherently nonrelativistic and so there is no relativistic circuit theory. This should actually not be surprising since circuit theory does not use the concept of space at all and relativity is a theory of spacetime.



The issue is in the assumptions that circuit theory is based on. The three assumptions are: (1) there is no net charge on any circuit element, (2) there is no magnetic flux outside of any circuit element, (3) the circuit is small so that electromagnetic influences can be assumed to propagate instantaneously.



Assumption (3) is expressly prohibited by relativity, and since a current density in one frame is both a current and a charge density in another then (1) will generally not be satisfied in all frames. Therefore, it is fatally flawed to attempt to find a relativistic version of Ohms law based on the equations of circuit theory since those laws are inherently non relativistic.



However, you could take the form of Ohm’s law that is based directly on Maxwell’s equations: $mathbf J = sigma mathbf E$. This law is based on Maxwell’s equations, which are completely relativistic. Since we know how $mathbf J$ and $mathbf E$ both transform then we can calculate how $sigma$ transforms.






share|cite|improve this answer












$endgroup$














  • $begingroup$
    J and E change the same way. So sigma doesn't change, it means R change like length?
    $endgroup$
    – Semen Yurchenko
    May 29 at 20:09






  • 6




    $begingroup$
    I would not recommend going from 𝜎 to 𝑅. Circuit theory is not relativistic so it doesn’t make sense to find transformation laws for quantities in circuit theory. Just leave it as 𝜎. By the way, J does not transform like E.
    $endgroup$
    – Dale
    May 29 at 20:37













12














12










12







$begingroup$


So basically I have three variants, but professor that I need to find one result. Any ideas?




Circuit theory is inherently nonrelativistic and so there is no relativistic circuit theory. This should actually not be surprising since circuit theory does not use the concept of space at all and relativity is a theory of spacetime.



The issue is in the assumptions that circuit theory is based on. The three assumptions are: (1) there is no net charge on any circuit element, (2) there is no magnetic flux outside of any circuit element, (3) the circuit is small so that electromagnetic influences can be assumed to propagate instantaneously.



Assumption (3) is expressly prohibited by relativity, and since a current density in one frame is both a current and a charge density in another then (1) will generally not be satisfied in all frames. Therefore, it is fatally flawed to attempt to find a relativistic version of Ohms law based on the equations of circuit theory since those laws are inherently non relativistic.



However, you could take the form of Ohm’s law that is based directly on Maxwell’s equations: $mathbf J = sigma mathbf E$. This law is based on Maxwell’s equations, which are completely relativistic. Since we know how $mathbf J$ and $mathbf E$ both transform then we can calculate how $sigma$ transforms.






share|cite|improve this answer












$endgroup$




So basically I have three variants, but professor that I need to find one result. Any ideas?




Circuit theory is inherently nonrelativistic and so there is no relativistic circuit theory. This should actually not be surprising since circuit theory does not use the concept of space at all and relativity is a theory of spacetime.



The issue is in the assumptions that circuit theory is based on. The three assumptions are: (1) there is no net charge on any circuit element, (2) there is no magnetic flux outside of any circuit element, (3) the circuit is small so that electromagnetic influences can be assumed to propagate instantaneously.



Assumption (3) is expressly prohibited by relativity, and since a current density in one frame is both a current and a charge density in another then (1) will generally not be satisfied in all frames. Therefore, it is fatally flawed to attempt to find a relativistic version of Ohms law based on the equations of circuit theory since those laws are inherently non relativistic.



However, you could take the form of Ohm’s law that is based directly on Maxwell’s equations: $mathbf J = sigma mathbf E$. This law is based on Maxwell’s equations, which are completely relativistic. Since we know how $mathbf J$ and $mathbf E$ both transform then we can calculate how $sigma$ transforms.







share|cite|improve this answer















share|cite|improve this answer




share|cite|improve this answer








edited May 29 at 20:23

























answered May 29 at 19:52









DaleDale

11.1k3 gold badges15 silver badges46 bronze badges




11.1k3 gold badges15 silver badges46 bronze badges














  • $begingroup$
    J and E change the same way. So sigma doesn't change, it means R change like length?
    $endgroup$
    – Semen Yurchenko
    May 29 at 20:09






  • 6




    $begingroup$
    I would not recommend going from 𝜎 to 𝑅. Circuit theory is not relativistic so it doesn’t make sense to find transformation laws for quantities in circuit theory. Just leave it as 𝜎. By the way, J does not transform like E.
    $endgroup$
    – Dale
    May 29 at 20:37
















  • $begingroup$
    J and E change the same way. So sigma doesn't change, it means R change like length?
    $endgroup$
    – Semen Yurchenko
    May 29 at 20:09






  • 6




    $begingroup$
    I would not recommend going from 𝜎 to 𝑅. Circuit theory is not relativistic so it doesn’t make sense to find transformation laws for quantities in circuit theory. Just leave it as 𝜎. By the way, J does not transform like E.
    $endgroup$
    – Dale
    May 29 at 20:37















$begingroup$
J and E change the same way. So sigma doesn't change, it means R change like length?
$endgroup$
– Semen Yurchenko
May 29 at 20:09




$begingroup$
J and E change the same way. So sigma doesn't change, it means R change like length?
$endgroup$
– Semen Yurchenko
May 29 at 20:09




6




6




$begingroup$
I would not recommend going from 𝜎 to 𝑅. Circuit theory is not relativistic so it doesn’t make sense to find transformation laws for quantities in circuit theory. Just leave it as 𝜎. By the way, J does not transform like E.
$endgroup$
– Dale
May 29 at 20:37




$begingroup$
I would not recommend going from 𝜎 to 𝑅. Circuit theory is not relativistic so it doesn’t make sense to find transformation laws for quantities in circuit theory. Just leave it as 𝜎. By the way, J does not transform like E.
$endgroup$
– Dale
May 29 at 20:37


















draft saved

draft discarded















































Thanks for contributing an answer to Physics Stack Exchange!


  • Please be sure to answer the question. Provide details and share your research!

But avoid


  • Asking for help, clarification, or responding to other answers.

  • Making statements based on opinion; back them up with references or personal experience.

Use MathJax to format equations. MathJax reference.


To learn more, see our tips on writing great answers.




draft saved


draft discarded














StackExchange.ready(
function ()
StackExchange.openid.initPostLogin('.new-post-login', 'https%3a%2f%2fphysics.stackexchange.com%2fquestions%2f483190%2frelativistic-resistance-transformation%23new-answer', 'question_page');

);

Post as a guest















Required, but never shown





















































Required, but never shown














Required, but never shown












Required, but never shown







Required, but never shown

































Required, but never shown














Required, but never shown












Required, but never shown







Required, but never shown









Popular posts from this blog

Tamil (spriik) Luke uk diar | Nawigatjuun

Align equal signs while including text over equalitiesAMS align: left aligned text/math plus multicolumn alignmentMultiple alignmentsAligning equations in multiple placesNumbering and aligning an equation with multiple columnsHow to align one equation with another multline equationUsing \ in environments inside the begintabularxNumber equations and preserving alignment of equal signsHow can I align equations to the left and to the right?Double equation alignment problem within align enviromentAligned within align: Why are they right-aligned?

Where does the image of a data connector as a sharp metal spike originate from?Where does the concept of infected people turning into zombies only after death originate from?Where does the motif of a reanimated human head originate?Where did the notion that Dragons could speak originate?Where does the archetypal image of the 'Grey' alien come from?Where did the suffix '-Man' originate?Where does the notion of being injured or killed by an illusion originate?Where did the term “sophont” originate?Where does the trope of magic spells being driven by advanced technology originate from?Where did the term “the living impaired” originate?